(25 intermediate revisions by 2 users not shown)
Line 1: Line 1:
<br>
+
[[Category:ECE]]
 +
[[Category:QE]]
 +
[[Category:CNSIP]]
 +
[[Category:problem solving]]
 +
[[Category:automatic control]]
 +
[[Category:optimization]]
 +
 
 +
= [[ECE PhD Qualifying Exams|ECE Ph.D. Qualifying Exam]] in "Automatic Control" (AC)  =
 +
 
 +
=  [[ECE-QE_AC3-2011|Question 3, August 2011]],  Part 3  =
  
= [[ECE PhD Qualifying Exams|ECE Ph.D. Qualifying Exam]]: Automatic Control (AC)- Question 3, August 2011 =
+
:[[ECE-QE AC3-2011 solusion-1|Part 1]],[[ECE-QE AC3-2011 solusion-2|2]],[[ECE-QE_AC3-2011_solusion-3|3]],[[ECE-QE AC3-2011 solusion-4|4]],[[ECE-QE AC3-2011 solusion-5|5]]
  
 
----
 
----
Line 9: Line 18:
 
<span class="texhtml">maximize &nbsp; &nbsp;−&nbsp;''x''<sub>1</sub> − 3''x''<sub>2</sub> + 4''x''<sub>3</sub></span><br>  
 
<span class="texhtml">maximize &nbsp; &nbsp;−&nbsp;''x''<sub>1</sub> − 3''x''<sub>2</sub> + 4''x''<sub>3</sub></span><br>  
  
<span class="texhtml"><sub></sub></span>subject to &nbsp;&nbsp;<span class="texhtml">''x''<sub>1</sub> + 2''x''<sub>2</sub> − ''x''<sub>3</sub> = 5</span>  
+
<span class="texhtml"><sub></sub></span>subject to &nbsp; &nbsp;<span class="texhtml">''x''<sub>1</sub> + 2''x''<sub>2</sub> − ''x''<sub>3</sub> = 5</span>  
  
 
<span class="texhtml">&nbsp; &nbsp; &nbsp; &nbsp; &nbsp; &nbsp; &nbsp; &nbsp; &nbsp; &nbsp;2''x''<sub>1</sub> + 3''x''<sub>2</sub> − ''x''<sub>3</sub> = 6</span>  
 
<span class="texhtml">&nbsp; &nbsp; &nbsp; &nbsp; &nbsp; &nbsp; &nbsp; &nbsp; &nbsp; &nbsp;2''x''<sub>1</sub> + 3''x''<sub>2</sub> − ''x''<sub>3</sub> = 6</span>  
  
&nbsp; &nbsp; &nbsp; &nbsp; &nbsp; &nbsp; &nbsp; &nbsp; &nbsp; &nbsp;<math>x_{1} \text{ free, } x_{2}\geq0, x_{3}\leq0.</math>
+
&nbsp; &nbsp; &nbsp; &nbsp; &nbsp; &nbsp; &nbsp; &nbsp; &nbsp; &nbsp;<math>x_{1} \text{ free, } x_{2}\geq0, x_{3}\leq0.</math>  
 +
 
 +
----
 +
 
 +
'''Theorem:'''
 +
 
 +
The '''Fundamental Theorem of Linear Programming''' that one of the basic feasible solutions is an optimal solution.&nbsp;
 +
 
 +
----
  
 
<math>\color{blue}\text{Solution 1:}</math>  
 
<math>\color{blue}\text{Solution 1:}</math>  
  
<math>\Rightarrow  x_{1}=5-2x_{2}+x_{3}=3-\frac{3}{2}x_{2}+\frac{1}{2}x_{3}</math>  
+
<math>\left.\begin{matrix}
 +
x_{1}+2x_{2}-x_{3}=5 \\
 +
2x_{1}+3x_{2}-x_{3}=6
 +
\end{matrix}\right\}\Rightarrow  x_{1}=5-2x_{2}+x_{3}=3-\frac{3}{2}x_{2}+\frac{1}{2}x_{3}</math>  
  
 
<math>\Rightarrow x_{2}-x_{3}=4</math>  
 
<math>\Rightarrow x_{2}-x_{3}=4</math>  
  
<math>\text{It is equivalent to  min }  x_{1}+3x_{2}-4x_{3}
+
<span class="texhtml">It is equivalent to  min ''x''<sub>1</sub> + 3''x''<sub>2</sub> − 4''x''<sub>3</sub> = 5 2''x''<sub>2</sub> + ''x''<sub>3</sub> + 3''x''<sub>2</sub> − 4''x''<sub>3</sub> = ''x''<sub>2</sub> − 3''x''<sub>3</sub> + 5,</span>
=5-2x_{2}+x_{3}+3x_{2}-4x_{3} = x_{2}-3x_{3}+5, x_{2}\geq0, x_{3}\leq0</math>  
+
 
 +
&nbsp; &nbsp; &nbsp; &nbsp; &nbsp; &nbsp; &nbsp; &nbsp; &nbsp; &nbsp; &nbsp; &nbsp; &nbsp; &nbsp; &nbsp; &nbsp;&nbsp;<math>x_{2}\geq0, x_{3}\leq0</math>  
  
<math>x_{2}-3x_{3}+5 = x_{2}-x_{3}-2x_{3}+5=9-2x_{3}\geq9</math>  
+
<math>x_{2}-3x_{3}+5 = x_{2}-x_{3}-2x_{3}+5=9-2x_{3}\geq9</math>&nbsp; &nbsp;<math>\color{green}  \text{constrain:  } x_{3}\leq0 \Rightarrow x_{3}=0</math>  
  
 
<math>\text{Equivalently, } -x_{1}-3x_{2}+4x_{3}\leq-9</math>  
 
<math>\text{Equivalently, } -x_{1}-3x_{2}+4x_{3}\leq-9</math>  
  
<math>\text{Equality is satisfied when } x_{3}=0, x_{2} =4, x_{1}=5-2\times4=-3</math>  
+
<math>\text{Equality is satisfied when } x_{3}=0, x_{2} =4+0=4, x_{1}=5-2\times4=-3</math>  
  
 
<math>\Rightarrow \left\{\begin{matrix}
 
<math>\Rightarrow \left\{\begin{matrix}
Line 34: Line 55:
 
x_{2}=4\\  
 
x_{2}=4\\  
 
x_{3}=0
 
x_{3}=0
\end{matrix}\right.</math>  
+
\end{matrix}\right.</math>&nbsp; &nbsp; &nbsp; &nbsp; &nbsp;&nbsp;<math>\color{green} \text{The answer is correct.}</math>
 +
 
 +
<math>\color{green} \text{But, this solution is NOT using the Fundamental Theorem of LP.}</math>  
  
 
----
 
----
  
<math>\color{blue}\text{Solution 2:}</math>
+
<math>\color{blue}\text{Solution 2:}</math>
 +
 
 +
One of the basic feasible solution is an optimal solution.<br><font face="serif"></font><span class="texhtml">The equality constraints can be represented in the form ''A'''x'''''<b> = ''b'',</b></span>  
  
One of the basic feasible solution is an optimal solution.<br><font face="serif"></font><math>\text{The equality constraints can be represented in the form } Ax=b, A= \begin{bmatrix}
+
<math>A=\begin{bmatrix}
 +
1 & 2 & -1\\
 +
2 & 3 & -1
 +
\end{bmatrix}=\begin{bmatrix}
 
a_{1}&  
 
a_{1}&  
 
a_{2}&  
 
a_{2}&  
 
a_{3}
 
a_{3}
 +
\end{bmatrix}; b=\begin{bmatrix}
 +
5\\
 +
6
 
\end{bmatrix}</math>  
 
\end{bmatrix}</math>  
  
<math>\text{The fist basis candidate is } \begin{pmatrix}
+
<math>\text{The first basis candidate is } \begin{pmatrix}
 
a_{1} &  
 
a_{1} &  
 
a_{2}  
 
a_{2}  
 
\end{pmatrix}</math>  
 
\end{pmatrix}</math>  
  
<math>\left [ A|b \right ] =\begin{bmatrix}  
+
&nbsp; &nbsp; &nbsp; &nbsp;&nbsp;<math>\left [ A|b \right ] =\begin{bmatrix}  
 
1 & 2 & -1 & 5 \\  
 
1 & 2 & -1 & 5 \\  
 
2 & 3 & -1 & 6  
 
2 & 3 & -1 & 6  
Line 59: Line 90:
 
\end{bmatrix}</math>  
 
\end{bmatrix}</math>  
  
<math>x^{\left( 1 \right)}= \begin{bmatrix}
+
&nbsp; &nbsp; &nbsp; &nbsp;&nbsp;<math>x^{\left( 1 \right)}= \begin{bmatrix}
 
-3 & 4 & 0
 
-3 & 4 & 0
\end{bmatrix} \text{ is BFS. } f_{1}=-9</math>  
+
\end{bmatrix}^{T} \text{ is a BFS. } f_{1}=-9</math>  
  
 
<math>\text{The second basis candidate is } \begin{pmatrix}
 
<math>\text{The second basis candidate is } \begin{pmatrix}
Line 68: Line 99:
 
\end{pmatrix}</math>  
 
\end{pmatrix}</math>  
  
<math>\left [ A|b \right ] =\begin{bmatrix}  
+
&nbsp; &nbsp; &nbsp; &nbsp;&nbsp;<math>\left [ A|b \right ] =\begin{bmatrix}  
 
1 & 2 & -1 & 5 \\  
 
1 & 2 & -1 & 5 \\  
 
2 & 3 & -1 & 6  
 
2 & 3 & -1 & 6  
Line 76: Line 107:
 
\end{bmatrix}</math>  
 
\end{bmatrix}</math>  
  
<math>x^{\left( 2 \right)}= \begin{bmatrix}
+
&nbsp; &nbsp; &nbsp; &nbsp;&nbsp;<math>x^{\left( 2 \right)}= \begin{bmatrix}
 
0 & 1 & -3
 
0 & 1 & -3
\end{bmatrix} \text{ is BFS. } f_{2}=-15</math>  
+
\end{bmatrix}^{T} \text{ is a BFS. } f_{2}=-15</math>  
  
 
<math>\text{The third basis candidate is } \begin{pmatrix}
 
<math>\text{The third basis candidate is } \begin{pmatrix}
Line 85: Line 116:
 
\end{pmatrix}</math>  
 
\end{pmatrix}</math>  
  
<math>\left [ A|b \right ] =\begin{bmatrix}  
+
&nbsp; &nbsp; &nbsp; &nbsp;&nbsp;<math>\left [ A|b \right ] =\begin{bmatrix}  
 
1 & 2 & -1 & 5 \\  
 
1 & 2 & -1 & 5 \\  
 
2 & 3 & -1 & 6  
 
2 & 3 & -1 & 6  
Line 93: Line 124:
 
\end{bmatrix}</math>  
 
\end{bmatrix}</math>  
  
<math>x^{\left( 2 \right)}= \begin{bmatrix}
+
&nbsp; &nbsp; &nbsp; &nbsp;&nbsp;<math>x^{\left( 2 \right)}= \begin{bmatrix}
 
1 & 0 & -5
 
1 & 0 & -5
\end{bmatrix} \text{ is BFS. } f_{3}=-21</math>  
+
\end{bmatrix}^{T} \text{ is a BFS. } f_{3}=-21</math>  
  
<math>\because f_{1}>f_{2}>f_{3}</math>  
+
<math>\because f_{1}>f_{2}>f_{3} \text{ , where } f_{1}=-9 \text{ is maximal.}</math>  
  
 
<math>\therefore \text{The optimal solution is } x^{*}=\begin{bmatrix}
 
<math>\therefore \text{The optimal solution is } x^{*}=\begin{bmatrix}
 
-3 & 4 & 0  
 
-3 & 4 & 0  
 
\end{bmatrix}  \text{ with objective value } -9</math>  
 
\end{bmatrix}  \text{ with objective value } -9</math>  
 +
 +
<math>\color{green} \text{This solution use the Fundamental Theorem of Linear Programming. }</math>
 +
 +
<math>\color{green} \text{All possible basic feasible solutions are generated }</math>
 +
 +
&nbsp; &nbsp; &nbsp; &nbsp; &nbsp; &nbsp; &nbsp;&nbsp;<math>\color{green} \text{ and from which the optimal one is selected.}</math>
 +
 +
----
 +
 +
<math>\color{blue} \text{Related Problem: Solve the following linear programming problem,}</math>
 +
 +
minimize &nbsp;&nbsp;<span class="texhtml">3''x''<sub>1</sub> + ''x''<sub>2</sub> + ''x''<sub>3</sub></span>
 +
 +
subject to &nbsp;<span class="texhtml">''x''<sub>1</sub> + ''x''<sub>3</sub> = 4</span>
 +
 +
&nbsp; &nbsp; &nbsp; &nbsp; &nbsp; &nbsp; &nbsp; &nbsp; &nbsp;&nbsp;<span class="texhtml">''x''<sub>2</sub> − ''x''<sub>3</sub> = 2</span>
 +
 +
&nbsp; &nbsp; &nbsp; &nbsp; &nbsp; &nbsp; &nbsp; &nbsp; &nbsp;&nbsp;<math>x_{1}\geq0,x_{2}\geq0,x_{3}\geq0,</math>
 +
 +
<math>\color{blue}\text{Solution :}</math>
 +
 +
<span class="texhtml">The equality constraints can be represented in the form ''A'''x'''''<b> = ''b'',</b></span>'''<br> '''
 +
 +
<math>A= \begin{bmatrix}
 +
1 & 0 & 1\\
 +
0 & 1 & -1
 +
\end{bmatrix}=\begin{bmatrix}
 +
a_{1}&
 +
a_{2}&
 +
a_{3}
 +
\end{bmatrix}; b=\begin{bmatrix}
 +
4\\
 +
2
 +
\end{bmatrix}</math>
 +
 +
<math>\text{The first basis candidate is } \begin{pmatrix}
 +
a_{1} &
 +
a_{2}
 +
\end{pmatrix}</math><br>
 +
 +
&nbsp; &nbsp; &nbsp; &nbsp; &nbsp;<math>\text{The corresponding basic solution is } x^{\left( 1 \right)}= \begin{bmatrix}
 +
4 & 2 & 0
 +
\end{bmatrix}^{T} \text{ is a BFS.}</math><br>
 +
 +
&nbsp; &nbsp; &nbsp; &nbsp; &nbsp;<span class="texhtml">The objective function value is ''f''<sub>1</sub> = 14</span>
 +
 +
<math>\text{The second basis candidate is } \begin{pmatrix}
 +
a_{1} &
 +
a_{3}
 +
\end{pmatrix}</math><br>
 +
 +
&nbsp; &nbsp; &nbsp; &nbsp;&nbsp;<math>\text{The corresponding basic solution is } x^{\left( 2 \right)}= \begin{bmatrix}
 +
6 & 0 & -2
 +
\end{bmatrix}^{T} \text{, which is NOT a BFS.}</math>&nbsp; &nbsp; &nbsp; &nbsp;
 +
 +
<math>\text{The third basis candidate is } \begin{pmatrix}
 +
a_{2} &
 +
a_{3}
 +
\end{pmatrix}</math>
 +
 +
&nbsp; &nbsp; &nbsp; &nbsp; &nbsp;<math>\text{The corresponding basic solution is } x^{\left( 3 \right)}= \begin{bmatrix}
 +
0 & 6 & 4
 +
\end{bmatrix}^{T} \text{, which is a BFS.}</math>
 +
 +
&nbsp; &nbsp; &nbsp; &nbsp; &nbsp;<span class="texhtml">The objective function value is ''f''<sub>3</sub> = 10</span>
 +
 +
<font color="#ff0000"><span style="font-size: 17px;">'''<math>\therefore \text{ the optimal solution is } x^{ * }= \begin{bmatrix} 0 & 6 & 4 \end{bmatrix}^{T}</math>'''</span></font>
  
 
<br>  
 
<br>  
Line 107: Line 205:
 
----
 
----
  
[[ECE PhD Qualifying Exams|Back to ECE Qualifying Exams (QE) page]]  
+
Automatic Control (AC)- Question 3, August 2011
 +
 
 +
Go to
 +
 
 +
*Problem 1: [[ECE-QE AC3-2011 solusion-1|solutions and discussions]]
 +
*Problem 2: [[ECE-QE AC3-2011 solusion-2|solutions and discussions]]
 +
*Problem 3: [[ECE-QE_AC3-2011_solusion-3|solutions and discussions]]
 +
*Problem 4: [[ECE-QE AC3-2011 solusion-4|solutions and discussions]]
 +
*Problem 5: [[ECE-QE AC3-2011 solusion-5|solutions and discussions]]
 +
 
 +
----
  
[[Category:ECE]] [[Category:QE]] [[Category:Automatic_Control]] [[Category:Problem_solving]]
+
<br> [[ECE PhD Qualifying Exams|Back to ECE Qualifying Exams (QE) page]]

Latest revision as of 10:10, 13 September 2013


ECE Ph.D. Qualifying Exam in "Automatic Control" (AC)

Question 3, August 2011, Part 3

Part 1,2,3,4,5

 $ \color{blue}\text{3. } \left( \text{20 pts} \right) \text{ Solve the following linear program, } $

maximize    − x1 − 3x2 + 4x3

subject to    x1 + 2x2x3 = 5

                   2x1 + 3x2x3 = 6

                   $ x_{1} \text{ free, } x_{2}\geq0, x_{3}\leq0. $


Theorem:

The Fundamental Theorem of Linear Programming that one of the basic feasible solutions is an optimal solution. 


$ \color{blue}\text{Solution 1:} $

$ \left.\begin{matrix} x_{1}+2x_{2}-x_{3}=5 \\ 2x_{1}+3x_{2}-x_{3}=6 \end{matrix}\right\}\Rightarrow x_{1}=5-2x_{2}+x_{3}=3-\frac{3}{2}x_{2}+\frac{1}{2}x_{3} $

$ \Rightarrow x_{2}-x_{3}=4 $

It is equivalent to min x1 + 3x2 − 4x3 = 5 − 2x2 + x3 + 3x2 − 4x3 = x2 − 3x3 + 5,

                                $ x_{2}\geq0, x_{3}\leq0 $

$ x_{2}-3x_{3}+5 = x_{2}-x_{3}-2x_{3}+5=9-2x_{3}\geq9 $   $ \color{green} \text{constrain: } x_{3}\leq0 \Rightarrow x_{3}=0 $

$ \text{Equivalently, } -x_{1}-3x_{2}+4x_{3}\leq-9 $

$ \text{Equality is satisfied when } x_{3}=0, x_{2} =4+0=4, x_{1}=5-2\times4=-3 $

$ \Rightarrow \left\{\begin{matrix} x_{1}=-3\\ x_{2}=4\\ x_{3}=0 \end{matrix}\right. $          $ \color{green} \text{The answer is correct.} $

$ \color{green} \text{But, this solution is NOT using the Fundamental Theorem of LP.} $


$ \color{blue}\text{Solution 2:} $

One of the basic feasible solution is an optimal solution.
The equality constraints can be represented in the form Ax = b,

$ A=\begin{bmatrix} 1 & 2 & -1\\ 2 & 3 & -1 \end{bmatrix}=\begin{bmatrix} a_{1}& a_{2}& a_{3} \end{bmatrix}; b=\begin{bmatrix} 5\\ 6 \end{bmatrix} $

$ \text{The first basis candidate is } \begin{pmatrix} a_{1} & a_{2} \end{pmatrix} $

        $ \left [ A|b \right ] =\begin{bmatrix} 1 & 2 & -1 & 5 \\ 2 & 3 & -1 & 6 \end{bmatrix} = \cdots = \begin{bmatrix} 1 & 0 & 1 & -3 \\ 0 & 1 & -1 & 4 \end{bmatrix} $

        $ x^{\left( 1 \right)}= \begin{bmatrix} -3 & 4 & 0 \end{bmatrix}^{T} \text{ is a BFS. } f_{1}=-9 $

$ \text{The second basis candidate is } \begin{pmatrix} a_{2} & a_{3} \end{pmatrix} $

        $ \left [ A|b \right ] =\begin{bmatrix} 1 & 2 & -1 & 5 \\ 2 & 3 & -1 & 6 \end{bmatrix} = \cdots = \begin{bmatrix} 1 & 0 & 1 & -3 \\ 1 & 1 & 0 & 1 \end{bmatrix} $

        $ x^{\left( 2 \right)}= \begin{bmatrix} 0 & 1 & -3 \end{bmatrix}^{T} \text{ is a BFS. } f_{2}=-15 $

$ \text{The third basis candidate is } \begin{pmatrix} a_{1} & a_{3} \end{pmatrix} $

        $ \left [ A|b \right ] =\begin{bmatrix} 1 & 2 & -1 & 5 \\ 2 & 3 & -1 & 6 \end{bmatrix} = \cdots = \begin{bmatrix} 0 & 1 & 1 & 4 \\ 1 & 1 & 0 & 1 \end{bmatrix} $

        $ x^{\left( 2 \right)}= \begin{bmatrix} 1 & 0 & -5 \end{bmatrix}^{T} \text{ is a BFS. } f_{3}=-21 $

$ \because f_{1}>f_{2}>f_{3} \text{ , where } f_{1}=-9 \text{ is maximal.} $

$ \therefore \text{The optimal solution is } x^{*}=\begin{bmatrix} -3 & 4 & 0 \end{bmatrix} \text{ with objective value } -9 $

$ \color{green} \text{This solution use the Fundamental Theorem of Linear Programming. } $

$ \color{green} \text{All possible basic feasible solutions are generated } $

              $ \color{green} \text{ and from which the optimal one is selected.} $


$ \color{blue} \text{Related Problem: Solve the following linear programming problem,} $

minimize   3x1 + x2 + x3

subject to  x1 + x3 = 4

                  x2x3 = 2

                  $ x_{1}\geq0,x_{2}\geq0,x_{3}\geq0, $

$ \color{blue}\text{Solution :} $

The equality constraints can be represented in the form Ax = b,

$ A= \begin{bmatrix} 1 & 0 & 1\\ 0 & 1 & -1 \end{bmatrix}=\begin{bmatrix} a_{1}& a_{2}& a_{3} \end{bmatrix}; b=\begin{bmatrix} 4\\ 2 \end{bmatrix} $

$ \text{The first basis candidate is } \begin{pmatrix} a_{1} & a_{2} \end{pmatrix} $

         $ \text{The corresponding basic solution is } x^{\left( 1 \right)}= \begin{bmatrix} 4 & 2 & 0 \end{bmatrix}^{T} \text{ is a BFS.} $

         The objective function value is f1 = 14

$ \text{The second basis candidate is } \begin{pmatrix} a_{1} & a_{3} \end{pmatrix} $

        $ \text{The corresponding basic solution is } x^{\left( 2 \right)}= \begin{bmatrix} 6 & 0 & -2 \end{bmatrix}^{T} \text{, which is NOT a BFS.} $       

$ \text{The third basis candidate is } \begin{pmatrix} a_{2} & a_{3} \end{pmatrix} $

         $ \text{The corresponding basic solution is } x^{\left( 3 \right)}= \begin{bmatrix} 0 & 6 & 4 \end{bmatrix}^{T} \text{, which is a BFS.} $

         The objective function value is f3 = 10

$ \therefore \text{ the optimal solution is } x^{ * }= \begin{bmatrix} 0 & 6 & 4 \end{bmatrix}^{T} $



Automatic Control (AC)- Question 3, August 2011

Go to



Back to ECE Qualifying Exams (QE) page

Alumni Liaison

Questions/answers with a recent ECE grad

Ryne Rayburn